- PowerScore Staff
- Posts: 5972
- Joined: Mar 25, 2011
- Fri Mar 16, 2012 2:46 pm
#85537
Complete Question Explanation
(The complete setup for this game can be found here: lsat/viewtopic.php?f=307&t=9348)
The correct answer choice is (C)
This is a very difficult List question.
Answer choice (A) is incorrect because it violates the fourth rule.
Answer choice (B) is incorrect because it would violate the second rule: as discussed during the setup, because of that rule, J can never be inspected on Monday morning.
Answer choice (C) is the correct answer. This is not a violation of the fourth rule: as discussed during the setup, L can be inspected in the morning and Z can be inspected in the afternoon.
Answer choice (D) is incorrect because it cause a violation of the third rule: when J is inspected on Tuesday morning, then from the second rule G must be inspected on Monday, but with S already inspected on Monday, a violation ensues.
Answer choice (E) is incorrect because it ultimately causes a violation of the third rule. With S, L, and Z as the morning inspections, only V remains to be inspected on Wednesday afternoon. At that point, 4 of the 6 buildings have been inspected, leaving only G and J. Due to the second rule, G must then be inspected on Monday afternoon, and J on Tuesday afternoon. But, that results in G and S both being inspected on Monday, a violation of the third rule.
(The complete setup for this game can be found here: lsat/viewtopic.php?f=307&t=9348)
The correct answer choice is (C)
This is a very difficult List question.
Answer choice (A) is incorrect because it violates the fourth rule.
Answer choice (B) is incorrect because it would violate the second rule: as discussed during the setup, because of that rule, J can never be inspected on Monday morning.
Answer choice (C) is the correct answer. This is not a violation of the fourth rule: as discussed during the setup, L can be inspected in the morning and Z can be inspected in the afternoon.
Answer choice (D) is incorrect because it cause a violation of the third rule: when J is inspected on Tuesday morning, then from the second rule G must be inspected on Monday, but with S already inspected on Monday, a violation ensues.
Answer choice (E) is incorrect because it ultimately causes a violation of the third rule. With S, L, and Z as the morning inspections, only V remains to be inspected on Wednesday afternoon. At that point, 4 of the 6 buildings have been inspected, leaving only G and J. Due to the second rule, G must then be inspected on Monday afternoon, and J on Tuesday afternoon. But, that results in G and S both being inspected on Monday, a violation of the third rule.
Dave Killoran
PowerScore Test Preparation
Follow me on X/Twitter at http://twitter.com/DaveKilloran
My LSAT Articles: http://blog.powerscore.com/lsat/author/dave-killoran
PowerScore Podcast: http://www.powerscore.com/lsat/podcast/
PowerScore Test Preparation
Follow me on X/Twitter at http://twitter.com/DaveKilloran
My LSAT Articles: http://blog.powerscore.com/lsat/author/dave-killoran
PowerScore Podcast: http://www.powerscore.com/lsat/podcast/